- The Step 1 Newsletter
- Posts
- ☕️ Which of the following groups of drugs would be appropriate to treat this patient’s symptoms?
☕️ Which of the following groups of drugs would be appropriate to treat this patient’s symptoms?
☕️ Morning Quiz 115
Morning Quiz ☕️
(Difficulty level - Low )

A 49-year-old woman presents to the urgent care clinic with:
Chief complaint: Palpitations for the past few hours
Associated symptoms:
Denies chest pain, shortness of breath, and sweating
Appears anxious and worried
Medical History
Mild bronchial asthma
Uses inhaler only during attacks (last used a week ago)
Former smoker
Drinks a couple of beers on weekends
No other significant medical history
Vital Signs
Heart rate: 136/min
Respiratory rate: 16/min
Temperature: 37.6°C (99.7°F)
Blood pressure: 120/80 mm Hg
Physical Examination
Unremarkable
Next Step
An electrocardiogram (ECG) is ordered.

Question
Which of the following groups of drugs would be appropriate to treat this patient’s symptoms?
A. α1-receptor antagonist
B. Selective β1-receptor antagonist
C. Non-selective β-receptor antagonist
D. α-receptor agonist
E. β-receptor agonist

Correct Answer 🎯: B. Selective ß1-receptor antagonist
⚙️

Trigger (Cause of Palpitations)
⬇
Anxiety / Sympathetic Overdrive (no structural heart disease, no acute ischemia)
⬇
↑ Sympathetic Stimulation → ↑ Catecholamines (NE/Epi)
⬇
Activation of β₁-Receptors (Heart) → ↑ cAMP → ↑ Ca²⁺ Influx
⬇
↑ Myocardial Contractility & ↑ SA Node Firing Rate
⬇
Sinus Tachycardia (HR 136/min)
⬇
Palpitations (Perceived Rapid Heartbeat)

Pro-tip

Differential Table ⚖️

Practice Questions ⏳️
Most importantly solve Uworld with QIDs
898, 19577
Your Brother in This Struggle
Dr. Shoaib Ahmad
P.S… Whenever you're ready to Excel USMLE Step 1, I’d love to help!
Book a Power Hour with me—just reply with "Private" in the subject line and share:
- A bit about your current prep (struggles, goals, etc.)
- The biggest questions you’d like answered
Let’s make your study time high-yield!
